Kann die Quantenmechanik wirklich dasselbe sein wie die zugrunde liegende deterministische Theorie?

Ich bin verblüfft über die jüngsten Arbeiten von 't Hooft, die eine explizite Konstruktion für eine zugrunde liegende deterministische Theorie auf der Grundlage ganzer Zahlen geben, die auf experimentell zugänglichen Skalen nicht von der Quantenmechanik zu unterscheiden ist. Bedeutet das, dass es sich um deterministische Komplexität handelt, die sich als Quantenzufälligkeit tarnt?

http://arxiv.org/abs/1204.4926

Zum gleichen Thema: Diskretion und Determinismus in Superstrings arxiv.org/abs/arXiv:1207.3612

Antworten (4)

Ich denke, zumindest einige Leser sollten inzwischen bemerkt haben, dass viele dieser Argumente, insbesondere die erbärmlicheren, eher eine Frage der Formulierung als der Physik sind. Sobald Sie Ihr Modell einfach genug gemacht haben, können Sie alles auf alles abbilden. Das war jetzt mein Ausgangspunkt: Wenn ein System hinreichend trivial ist, kann man machen, was man will. Wie können wir nun einige dieser sehr einfachen Ergebnisse nachträglich zu etwas Interessanterem verallgemeinern?

Dies war die Grundregel meines Ansatzes. Ich interessiere mich überhaupt nicht für No-Go-Theoreme, mich interessiert die Frage "was kann man stattdessen tun?" Ich gebe zu, dass ich die Probleme des Universums nicht lösen kann, ich habe die Theorie von allem nicht gefunden. Anstatt pathetisch zu verkünden, was man nicht tun soll, versuche ich Schritt für Schritt Modelle zu konstruieren.

Ich denke, ich habe jetzt einige Modelle produziert, die es wert sind, diskutiert zu werden. Sie sind vielleicht noch nicht groß und kompliziert genug, um unser Universum zu beschreiben, aber sie könnten unsere Fragen bezüglich der Unterscheidung zwischen Quantenmechanik und klassischen Theorien in eine neue Perspektive rücken. Wenn ein System zu einfach ist, verschwindet diese Unterscheidung natürlich. Aber wie weit darf man gehen? Denken Sie daran, dass zellulare Automaten enorm komplex werden können, ebenso quantenmechanische Modelle. Wie weit können wir gehen, um die beiden in Beziehung zu setzen? So sollten Sie meine Papiere betrachten. Ich denke zufällig, dass die Frage sehr wichtig ist, und man kann viel weiter gehen, wenn es darum geht, Quantenmodelle mit klassischen Modellen in Beziehung zu setzen, als manche Leute uns glauben machen wollen.

Und ist eine Rechnung falsch, wenn jemandem die Formulierung nicht gefällt?

Ich stimme zu, dass die Modelle interessant und das Programm wichtig sind, aber das Hauptproblem besteht darin, dass Sie nicht gezeigt haben, dass die Quantenmechanik auf einer beable-Basis der klassischen Statistik auf einer CA entspricht, und dies möglicherweise nicht wahr ist (ich denke, es ist gilt ohne weitere Einschränkungen nicht). Ich werde eine direkte Frage stellen, da Ihre interessanten Antworten und Kommentare es einem ermöglicht haben, das eigene Denken darüber zu schärfen.

Aktuelle (experimentelle und theoretische) Erkenntnisse über deterministische Ansätze zum Quanten-Nichtdeterminismus besagen lediglich, dass jede deterministische Theorie, die der Quantenmechanik zugrunde liegt, nichtlokal sein muss. Die Forschung geht dann weiter, um die genaue Natur dieser Nichtlokalität zu diskutieren oder bestimmte Versionen auszuschließen.

Auf der anderen Seite gibt es diejenigen, die nichtlokale deterministische Theorien konstruieren, die sich irgendwie auf QM reduzieren. Viel Arbeit steckt in der Bohmschen Mechanik, die jedoch Schwierigkeiten hat, eine realistische Quantenfeldtheorie wiederzugewinnen.

Das Papier von t'Hooft verfolgt einen anderen Ansatz, der auf Diskretion basiert. Seine Ergebnisse sind jedoch derzeit sehr begrenzt und reproduzieren nur den harmonischen Oszillator.

Ihr erster Satz ist vielleicht zu kategorisch. So verstehe ich zum Beispiel, dass es keinen schlupflochfreien experimentellen Beweis für Nichtlokalität gegeben hat. Andererseits ist Nichtlokalität eine außergewöhnliche Behauptung, also bedarf es eines außergewöhnlichen Beweises.
t'Hooft "reproduziert" nichts - er hat einen linearen Raum, der ein Quantenzustandsraum ist, er verkürzt nur die Evolution, um auf einer speziellen Basis deterministisch zu sein. Die eigentliche Dynamik ist immer noch quantenhaft, es ist nicht wirklich ein Automat – Sie können sich nicht vorstellen, dass die Zustände darunter klassisch sind.
„Allerdings sind seine Ergebnisse derzeit sehr begrenzt und reproduzieren nur den harmonischen Oszillator.“ Er hat zwei Folgearbeiten, die die Äquivalenz einer bosonischen Feldtheorie in 1 + 1-Dimensionen und auch eine „Superstring“-ähnliche Lösung für das zugrunde liegende deterministische Gitter zeigen ( arxiv. org/abs/1207.3612 und arxiv.org/abs/1205.4107 ) Die Argumente dort scheinen mir jedoch eine Erweiterung der Argumente für das erste Papier zu sein. Ich konnte keine Lücke in der Zuordnung finden. Daher die Frage. Was genau verstößt das Schema?
Ich stimme zu, dass es nicht viele Ergebnisse gibt, insbesondere nichts, was das Modell von anderen unterscheiden würde. Vielen Dank für Ihren Kommentar. - Asche
@a25bedc5-3d09-41b8-82fb-ea6c353d75ae: (Bitte wählen Sie einen besseren Namen zur Identifizierung) - Ich habe mir die Details nicht angesehen, da ich bei all diesen Versuchen den Eindruck habe, dass sie Sackgassen sind; viele technische Einzelheiten, aber im Endeffekt nichts, was das Verständnis dessen, was wir bereits wissen, erweitert. Wenn der Quantenindeterminismus in der Natur aus einer zugrunde liegenden diskreten klassischen Dynamik erzeugt wird, wette ich, dass dies ein eleganter Weg ist, genau wie die Quantenmechanik eine elegante Variation der klassischen Mechanik ist, keine chaotische.
@akhmeteli: Partikel-Nichtlokalität ist eine gewöhnliche experimentelle Tatsache (bereits in Interferenzexperimenten sichtbar); nur die genaue Form, die sie in der Theorie annimmt, ist zweifelhaft. Ich stimme also nicht zu, dass es einer außergewöhnlichen Überprüfung bedarf. Die philosophischen Probleme verschwinden, sobald man das Teilchenbild aufgibt; dann gibt es nichts Widersinniges mehr, das außerordentlicher Aufmerksamkeit bedürfe.
@Arnold Neumaier: Geht es bei dieser "Partikel-Nichtlokalität" um deine persönliche Meinung oder könntest du einen Hinweis geben? Lassen Sie mich nur anmerken, dass quantenähnliche Teilcheninterferenz in einem klassischen System demonstriert wurde: PRL 97, 154101 (2006) (z. B. hekla.ipgp.fr/IMG/pdf/Couder-Fort_PRL_2006.pdf )
@akhmeteli: Geometrische Optik ist die klassische Teilchenansicht des klassischen Lichts und weist alle nichtlokalen Merkmale auf, die in der Quantenmechanik auftreten, sobald Sie über den Bereich der Gültigkeit der Annäherung hinausgehen. Dies sagt mir (obwohl es nicht der Mainstream-Standpunkt ist), dass die Wurzel der Quanten-Nichtlokalität darin besteht, Quantenobjekte als Teilchen und nicht als Feldanregungen zu behandeln. Im Kontext der relativistischen Quantenfeldtheorie ist alles lokal, was zeigt, dass die seltsamen Aspekte nichts als eine schlechte Wahl der intuitiven Visualisierung sind.
@Arnold Neumaier: Ich kann die geometrische Optik nicht als Beweis für "Teilchen-Nichtlokalität" akzeptieren - Sie könnten das Coulomb-Gesetz mit gleichem "Erfolg" erwähnen - ja, es ist nichtlokal, aber es ist nur eine Annäherung, die in Experimenten genau dort versagt, wo es sagt Nichtlokalität voraus. Wir sind uns also nicht einig, ob Nichtlokalität eine außergewöhnliche Behauptung ist, und Sie haben keinen Hinweis gegeben, um Ihren Standpunkt zu stützen. Darüber hinaus bestehe ich darauf, dass es aufgrund von Schlupflöchern bisher keinen positiven experimentellen Beweis für Nichtlokalität in der Quantentheorie gibt.
@akhmeteli: Alle Physik ist nur eine Annäherung, die fehlschlägt, wenn Sie die Situation genauer beschreiben. Und es gibt keine positiven experimentellen Beweise für irgendetwas Mikroskopisches, wenn man so sorgfältig nach Schlupflöchern sucht wie in der Literatur über Nichtlokalität.
Lokalität=>Determinismus (siehe JS Bell in „Sixty-two years of uncertainty“).
@Arnold Neumaier: Klar :-) Warum werden dann in den besten Fachzeitschriften gerade experimentelle Arbeiten zur Schlupflochbeseitigung veröffentlicht, wenn die Schlupflöcher so unbedeutend sind, wie Sie es beschreiben? Warum so ein fieberhaftes Rennen unter den besten Experimentatoren, die versuchen, die Schlupflöcher zu schließen? Zeilinger und Co-Autoren schrieben kürzlich: „Die Bellsche Ungleichung zu verletzen und gleichzeitig all diese Schlupflöcher zu schließen, ist heute eine der wichtigsten noch offenen Herausforderungen in der Grundlagenphysik.“ (PNAS, 16. November 2010, Band 107, Nr. 46, 19708–19713). Deshalb habe ich gesagt, dass Ihre Zusammenfassung vielleicht zu kategorisch ist:-)
@quantropy: ''Lokalität=>Determinismus'' nur unter einigen Annahmen, die in der Quantenfeldtheorie nicht erfüllt sind.
@akhmeteli: Man kann nie alle Schlupflöcher schließen. Das liegt in der Natur experimenteller Arbeit.
@Arnold Neumaier: Sie geben keine Referenzen an, und ich bin mir nicht sicher, ob dies eine Mainstream-Ansicht ist - z. B. scheint Zeilinger in diesem speziellen Fall damit nicht einverstanden zu sein (was, wie gesagt, darauf hinweist, dass Sie tatsächlich zu kategorisch die Ansicht von Experten beschrieben haben der Versuchssituation). Es kann gut sein, dass Sie in diesem speziellen Fall Recht haben und die Verstöße nicht nachgewiesen werden können, wenn alle Schlupflöcher geschlossen sind, aber das würde bedeuten, dass die Schlupflöcher tatsächlich entscheidend und nicht unbedeutend sind, wie Sie zu argumentieren scheinen.
Ich habe nie behauptet, dass alle meine Antworten Mainstream sind. Trotzdem sind sie normalerweise richtig (und ich korrigiere sie, wenn sie sich als falsch erweisen). Die Wahrheit hängt nicht von der Anzahl der Neinsager ab.

Es ist durchaus möglich, dass QM auf einem deterministischen physikalischen Mechanismus beruht. Die No-Go-Theoreme wie das Bellsche Theorem oder das "Theorem des freien Willens" von Conway und Kochen sind gegen deterministische Theorien über verborgene Variablen nicht wirksam, da sie Nichtdeterminismus als eine ihrer Annahmen erfordern. Es gibt immer noch viele Physiker, die behaupten, der Determinismus sei widerlegt, aber sie begehen den logischen Irrtum. Es ist jedoch noch zu früh zu sagen, ob 't Hooft auf dem richtigen Weg ist.

-1: Das ist falsch. Das Bell-Theorem ist wirksam gegen deterministische Theorien über verborgene Variablen, die lokal sind. Das Argument des Superdeterminismus ist lächerlich, es ist keine Wissenschaft, es ist Verschwörungstheorie.

t'Hoofts Papiere sind ungültig. Sie machen einen Fehler, nämlich dass sie davon ausgehen, dass die Quantentheorie eine klassische Theorie ist, nur weil der diskrete Zeitentwicklungsoperator in einem Quantensystem auf irgendeiner Basis eine Permutation ist.

t'Hooft betrachtet diskrete Zeitquantensysteme, bei denen die Zeitentwicklung in gewisser Weise eine diskrete Permutation ist. Wenn Sie also ein 3-Zustandssystem haben, vertauschen Sie 1 zu 2 zu 3. Er analysiert dann den Raum aller Überlagerungen dieser drei Zustände und entdeckt, dass er die Quantenmechanik wiederherstellen kann. Er erklärt dann, dass "die Quantenmechanik einem klassischen deterministischen System entspricht".

Das ist einfach falsch. Ich nehme an, t'Hooft denkt, wenn Sie in einem Basiszustand beginnen, bleiben Sie für immer in einem Basiszustand, indem Sie nur den Basiszustand permutieren, und daher muss dies ein klassisches deterministisches System sein. Aber der Punkt ist, dass der Zustandsraum alle Arten von Quantenüberlagerungen der Basiszustände enthält, und diese anderen Zustände , die Nicht-Basiszustände, sind Überlagerungen nicht durch klassische Wahrscheinlichkeit, sondern durch Quantenamplituden.

Wenn Sie Quantenamplituden haben, selbst wenn sich die Basiszustände durch Permutation entwickeln, kann die Theorie offensichtlich die Quantenmechanik reproduzieren, weil sie Quantenmechanik ist.

Tatsächlich gibt es hier ein Theorem: Zu jedem endlichdimensionalen quantenmechanischen Hamiltonoperator H existiert ein Permutationssystem, das diesen Hamiltonoperator in einer Näherung enthält und auf einen Unterraum der Zustände wirkt.

Der Beweis: Diagonalisiere H zu einer N mal N Diagonalmatrix mit N Eigenwerten und approximiere die N Energien durch rationale Zahlen mit enormen Primzahlen, p ich / q ich 1 < ich < N , und nehmen Sie einen Einheitszeitschritt. Multipliziere alle q_i miteinander und nenne das Produkt Q. Dann ist die Exponentialfunktion von t mal Hamitlonian periodisch mit Periode Q Zeitschritten.

Betrachten Sie nun einen Zustandsraum, dessen Basis durch ein N-Tupel ganzer Zahlen von 1 bis Q gekennzeichnet ist. Der Permutations-Hamiltonoperator nehme das Basiselement (a_1,....a_n) zu a ich a ich + s ich wo s ich ist das Produkt aller q außer q_i und der Z q ich multiplikative Umkehrung von p ich . Dieser Permutations-Hamiltonoperator muss die Eigenschaft haben, dass seine Eigenwerte eine Teilmenge enthalten p ich / q ich . Projizieren Sie in diesen Unterraum und nennen Sie dies Ihr Quantensystem.

Dieser Prozess oder etwas Ähnliches kann in keiner Weise als "deterministisches System" bezeichnet werden. Es gibt immer noch Zustände, die Superpositionen sind. Wenn Sie ein echtes klassisches System haben, wird der Zustand durch eine Wahrscheinlichkeitsverteilung für den unbekannten Anfangszustand beschrieben, nicht durch Wahrscheinlichkeitsamplituden für Überlagerungen des unbekannten aktuellen Zustands. In dem Moment, in dem Sie Zustände durch Überlagerungen beschreiben, holen Sie die Quantenmechanik nicht heraus, Sie setzen sie ein.

Aus diesem Grund ist t'Hooft in der Lage, mathematische Ergebnisse abzuleiten, die quantenmechanisch sind, er verwendet die Quantenmechanik, aber mit der Einschränkung, dass sie sich auf eine Permutation auf einer Basis reduziert. Das erklärt nicht, warum wir Überlagerungen elektronischer Spins in der Natur sehen, sie erzeugt diese Überlagerungen nicht aus Unkenntnis klassischer Werte, sie fügt die Überlagerungen von Hand ein.

Ich mag t'Hoofts Motivation und bewundere sein unabhängiges Denken, aber das ist kein valides Zeug. Es tut nicht das, was er behauptet. Die Behauptung, es handele sich um klassische Modelle, als irreführend zu bezeichnen, ist gemeinnützig.

't Hooft glaubt, dass es eine ontologisch bevorzugte Basis gibt und Überlagerungen dieser Basiszustände nicht real sind. Die bohmsche Behandlung von Messwechselwirkungen zeigt, dass Sie eine bevorzugte Basis haben und dennoch andere Observable korrekt beschreiben können - obwohl die Bohmschen auch immer noch Überlagerungen in ihrer Ontologie als Zustände der Pilotwelle haben (es sei denn, sie nehmen den "nomologischen" Weg und behandeln die spezifische Pilotwelle eines individuellen Systems eher als dynamisches Gesetz als als physikalischer Zustand)...
@MitchellPorter: Ich weiß, was er sagt , aber das ist lächerlich --- Sie müssen mir sagen, warum ein Laborelektron durch eine Überlagerung beschrieben wird. Es hilft nicht zu sagen, dass es eine Basis gibt, in der der Hamiltonoperator eine Permutation ist. Wenn wir nicht wissen, in welchem ​​Basiselement sich unser Universum befindet, beschreiben wir das mit einer Wahrscheinlichkeitsverteilung, nicht mit Amplituden. Dann gibt es keinen Grund dafür, dass das Elektron im Labor mit Amplituden beschrieben wird. Es ist einfach falsch.
In der Bohmschen Mechanik mit einer bestimmten Pilotwelle kann man die spezifische Pilotwelle in die Bewegungsgleichungen für die klassischen Objekte der Theorie einsetzen und erhält am Ende eine pseudoklassische Theorie, in der eine klassische Bewegungsgleichung um a erweitert wird nichtlokales Potenzial. Es muss möglich sein, die Überlagerung in Bezug auf dieses nichtlokale Potential zu erklären, weil die Theorie immer noch identisch ist mit der Bohmschen Mechanik mit einer bestimmten Pilotwelle, aber niemand hat jemals diesen Weg genommen und gezeigt, wie solche Erklärungen aussehen ...
In dem Artikel nach diesem konstruiert 't Hooft seine angebliche Zuordnung von einer CA zu einer QFT. Die Zertifizierungsstelle ist wirklich einfach, aber das Mapping ist ein wenig nicht trivial; zumindest habe ich die Essenz davon noch nicht begriffen. Es wird schwierig sein, irgendetwas Konkretes darüber zu sagen, wie oder ob 't Hooft beobachtete Überlagerungen erklären kann, bis jemand diese weitere Phase seiner jüngsten Arbeit versteht.
@MitchellPorter: Ich habe lange gebraucht, um es zu verstehen, weil es eindeutig falsch ist, und ich habe versucht, eine echte Karte von CA zu QM zu erstellen. Was er tut, ist das, was ich beschrieben habe – er nimmt ein QM-System und wandelt es in einen Fall um, in dem es sich auf einer Basis in eine Permutation verwandelt, und wenn er dies tun kann (was immer der Fall ist), erklärt er, dass er QM herausgeholt hat eines klassischen Automaten. Die Deklaration ist falsch, das Verfahren erzeugt einen "t'Hooft-Quantenautomaten", keinen klassischen Automaten, da es a priori Überlagerungszustände enthält.
Die neue Version v3 meines Papers quant-ph/ 1405.1548 wurde heute, 21.12.2015, an das ArXiv gesendet. „The Cellular Automaton Interpretation of Quantum Mechanics“. Meine Antwort auf die hier gestellte Frage lautet: ja. Kritik ist immer willkommen, auch wenn es von Laien kommt, obwohl ich eine Antwort nicht garantieren kann.